PDA

توجه ! این یک نسخه آرشیو شده میباشد و در این حالت شما عکسی را مشاهده نمیکنید برای مشاهده کامل متن و عکسها بر روی لینک مقابل کلیک کنید : سوالات و اشكالات ریاضی



Only Math
3rd November 2010, 11:35 AM
سلام
دوستان خوبم لطفا سوالات و اشکالات ریاضی خودتون رو تو این بخش بپرسین تا هم جواب رو سریع تر بدست بیارید و هم دوستان دیگه ای که اون مشکل رو دارن از پاسخها استفاده کنن.
ممنون [golrooz]

بانوثریا
3rd November 2010, 02:54 PM
http://njavan.com/forum/showthread.php?t=79866
سلام خدا خیرتون بده اینا رو جواب بدید ممنون

azarbara
11th November 2010, 12:37 PM
دوستان برای اینکه سرگرم بشید این سوال رو حل کنید و اگه حل کردید فقط اعلام کنید که حل کردید یا نه

سرآخر جوابا رو می ذاریم که همه استفاده کنن


file:///C:/Users/p/AppData/Local/Temp/msohtmlclip1/01/clip_image002.gif

azarbara
14th November 2010, 08:19 AM
من خودم اعلام می کنم که حل کردم (البته 3 سال پیش)
دوستان دست به قلم بشید

mohebbi
14th November 2010, 12:27 PM
حدود نداره این انتگرال شما؟

azarbara
14th November 2010, 12:41 PM
حدود نداره این انتگرال شما؟
نه انتگرال نامعین هست.

Only Math
14th November 2010, 02:59 PM
منم حلش کردم

azarbara
16th November 2010, 01:32 PM
دوستان سوالات بعدی در راهه یه یا علی بگید و سوالاتو حل کنید

azarbara
16th November 2010, 08:10 PM
یه سوال دیگه
e (عدد نپر) به توان پی بزرگتره یا پی به توان e (لطفا بدون مراجعه به ماشین حساب)
راجع به این موضوع هم نظر بدید که یه عدد دلخواه به توان یه عدد اصم رو چطور می شه حساب کرد؟

بانوثریا
16th November 2010, 08:41 PM
اگه رياضيتون انقد خوبه اون سوالاي منو جواب بديد

azarbara
18th November 2010, 12:43 AM
اگه رياضيتون انقد خوبه اون سوالاي منو جواب بديد

با سلام خدمت شما ما هیچ ادعایی نداریم
ولی من چند بار خواستم سوالای شما رو ببینم روش کلیک می کنی می نویسه این مطلب وجود ندارد و یا حذف شده است شما لطف کن و سوالاتو یه بار دیگه بذار شاید بشه کاری کرد

بانوثریا
19th November 2010, 02:11 PM
tnx[golrooz]

azarbara
20th November 2010, 01:21 PM
tnx[golrooz]
این فایل rar از حالت فشرده خارج نشد یا فایل مشکل داره یا winrar من که احتمالش خیلی زیاده که فایل مشکل داشته باشه اگه امکانش هست سوال رو همین جا بنویسید تا ببینیم چیکار میشه کرد!
و در ضمن اعلام کنید که سوال در چه زمینه ای هست!

azarbara
21st November 2010, 11:42 AM
این فایل rar از حالت فشرده خارج نشد یا فایل مشکل داره یا winrar من که احتمالش خیلی زیاده که فایل مشکل داشته باشه اگه امکانش هست سوال رو همین جا بنویسید تا ببینیم چیکار میشه کرد!
و در ضمن اعلام کنید که سوال در چه زمینه ای هست!
سوالارو دریافت کردم روش فکر می کنم و نتیجه رو به شما اطلاع میدم فقط امیدوارم که نتیجه مثبت باشه

*مینا*
22nd November 2010, 07:51 PM
سلام
چند تا سوال خیلی ساده دارم ( خودم یادم رفته ) اگه کمک کنید ممنون می شم


Sin(4/3)pi - Sin (2/3) pi

pi = عدد پی [nishkhand]

بعد یه چیز دیگه که یادم رفته اینه که ، وقتی دو عبارت که با هم ضرب شدن و می خوایم مشتق بگیریم ، میگیم ، مشتق اولی در دومی + مشتق دومی در اولی ؛ یا منها ؟

بعد یه سوال دیگه هم دارم ، این که کسینوس (به توان دو )پی جوابش چی میشه ؟

Only Math
22nd November 2010, 08:28 PM
سلام
چند تا سوال خیلی ساده دارم ( خودم یادم رفته ) اگه کمک کنید ممنون می شم


Sin(4/3)pi - Sin (2/3) pi

pi = عدد پی [nishkhand]
سلام
الان منظور از Sin(4/3)pi - Sin (2/3) pi اینه که pi جز کمانه یا ضریب ؟




بعد یه چیز دیگه که یادم رفته اینه که ، وقتی دو عبارت که با هم ضرب شدن و می خوایم مشتق بگیریم ، میگیم ، مشتق اولی در دومی + مشتق دومی در اولی ؛ یا منها ؟
مثبت



بعد یه سوال دیگه هم دارم ، این که کسینوس (به توان دو )پی جوابش چی میشه ؟
اگر منظور عدد باشه کافیه کسینوس پی رو حساب کنید ( برابر با 1- ) بعد به توان 2 برسونید که میشه 1
اگر منظور عبارت متناظر با کسینوس (به توان دو )پی باشه جواب میشه :
1 منهای سینوس(به توان دو) پی که بازم برابر با 1 خواهد بود ( 1 =0 - 1)

مهندس هاله
22nd November 2010, 08:43 PM
سلام
الان منظور از sin(4/3)pi - sin (2/3) pi اینه که pi جز کمانه یا ضریب ؟



منها



مهندس منها؟؟؟؟؟؟؟؟؟؟؟؟؟؟؟؟؟؟؟؟؟ ؟؟؟؟؟؟؟؟
ابرومونو بردي!
خودمم يه لحظه شك كردم!

*مینا*
22nd November 2010, 08:50 PM
مرسی [golrooz]


اگر منظور عدد باشه کافیه کسینوس پی رو حساب کنید ( برابر با 1- ) بعد به توان 2 برسونید که میشه 1
اگر منظور عبارت متناظر با کسینوس (به توان دو )پی باشه جواب میشه :
1 منهای سینوس(به توان دو) پی که بازم برابر با 1 خواهد بود ( 1 =0 - 1) این درست
طبق رابطه ::

http://uc-njavan.ir/images/86qjafeq8g0uk5en30ju.jpg


جواب این چرا 1/2 میآد ؟!
یا من اشتباه می کنم :-??


منهادر مورد این مطمعنی؟ !! فک کنم باید مثبت باشه ها

مهندس هاله
22nd November 2010, 09:07 PM
مينا جان احتمالا بايد رابطه ايراد داشته باشه!
مطمئني درسته؟

*مینا*
22nd November 2010, 09:14 PM
مينا جان احتمالا بايد رابطه ايراد داشته باشه!
مطمئني درسته؟
سلام
یه دور دیگه نگاه کردم ، محض احتیاط
ولی رابطه همینه ....

Only Math
22nd November 2010, 09:31 PM
مرسی [golrooz]

در مورد این مطمعنی؟ !! فک کنم باید مثبت باشه ها


مهندس منها؟؟؟؟؟؟؟؟؟؟؟؟؟؟؟؟؟؟؟؟؟ ؟؟؟؟؟؟؟؟
ابرومونو بردي!
خودمم يه لحظه شك كردم!

واقعا ببخشید اشتباه شد [khejalat]
مثبته! ويرايش كردم [nishkhand]



طبق رابطه ::


http://uc-njavan.ir/images/86qjafeq8g0uk5en30ju.jpg


جواب این چرا 1/2 میآد ؟!
یا من اشتباه می کنم :-??


نه جواب 1 میشه:
کسینوس دو پی برابر با 1 پس جواب پرانتز میشه صفر در نتیجه جواب میشه 1 [golrooz]

*مینا*
22nd November 2010, 09:41 PM
نه جواب 1 میشه:
کسینوس دو پی برابر با 1 پس جواب پرانتز میشه صفر در نتیجه جواب میشه 1 [golrooz]
این و قبول دارم
راستش ما تو تمریناتمون تعداد زیادیشون با رابطه ای که نوشتم حل می شه
ولی یه جایی به مشکل بر خوردم ، واسه همین کسینوس توان دو پی رو مثال زدم تا راحت بشه به جواب رسید ...
یعنی با حل کردن می رسیم به سینوس به توان دو (...) که تبدیلش می کنیم به همون رابطه ی بالا سپس ازش انتگرال می گیریم ،
اما یه جایی از تمرینم رسیدم به کسینوس به توان دو (...) که نیازی به انتگرال نداشت ...

پس تو همچین مواردی این طوری حل کنم ؟ فردا امتحان دارمآ [bamazegi]

azarbara
27th November 2010, 09:49 PM
tnx[golrooz]
اين جواب سوال چهاره رو بقيه هم دارم فكر مي كنم اميدوارم تا فردا حل بشن
البته قولي نميشه داد كه همش حل بشن
ولي اميدوارم حل شن؟
باتشکر از دوست عزیزم آقای طالعی

azarbara
27th November 2010, 10:06 PM
این و قبول دارم
راستش ما تو تمریناتمون تعداد زیادیشون با رابطه ای که نوشتم حل می شه
ولی یه جایی به مشکل بر خوردم ، واسه همین کسینوس توان دو پی رو مثال زدم تا راحت بشه به جواب رسید ...
یعنی با حل کردن می رسیم به سینوس به توان دو (...) که تبدیلش می کنیم به همون رابطه ی بالا سپس ازش انتگرال می گیریم ،
اما یه جایی از تمرینم رسیدم به کسینوس به توان دو (...) که نیازی به انتگرال نداشت ...

پس تو همچین مواردی این طوری حل کنم ؟ فردا امتحان دارمآ [bamazegi]
با سلام اگه اجازه بديد منم وارد بحث شما بشم
تو حل انتگرال كه شامل توابع مثلثاتي هست به قول استادمون ما بايد از شر توان دو و راديكال راحت شيم از شر توان دو با همين دو فرمول ميشه راحت شد
sinx]^2=(1-cos2x)/2
چون اينجا سخته نوشتنش ديگه رياضي نمي نويسم توضيح ميدم
كسينوس به توان دو هم مي شه همين كسر، با اين تفاوت كه تو صورت به جاي منها جمع مي شينه
فرضا اگه تو يه سوال توان سينوس يا كسينوس زوج شد مثلا 4 به جاي سينوس به توان 2 عبارت بالا رو مي نويسي و كل كسرو به توان دو مي رسوني كه برابر با سينوس به تان چهار بشه كه تو اين مرحله چون صورت اتحاد اول ميشه يه كسينوس به توان دو 2x ظاهر مي شه كه عمليات بالا رو يه بار ديگه براي خلاصي از شر اين توان مي نويسيم سپس انتگرال را محاسبه مي كنيم
البته ببخشيد كه من وارد بحث شما شدم
موفق باشيد

azarbara
28th November 2010, 01:25 PM
این یه راه حلی هست که به ذهنم رسید البته هنوز رو کاغذ پیادش نکردم امیدوارم همه چیز خوب پیش بره
شما می تونی ریشه های چهارم واحد رو محاسبه کنی که چهار تا عدد بدست میاد اونارو مساوی A,B,C,D بگیر
ریشه های n ام واحد روی دایره واحد قرار می گیرن و یک n ضلعی منتظم هم تشکیل میدن (هر دوی اینا قضیه هست که اثبات شده)
جمع ریشه های واحد هم صفر هست
پس اون چهار ضلعی که تو دایره محاط هست طبق قضایای فوق باید یک چهار ضلعی منتظم باشه پس يه مستطيل نيز هست

azarbara
29th November 2010, 04:30 PM
خسته نباشید
چون من امکان اسکن و حتی گرفتن عکس از کاغذ رو ندارم همین جا توضیح میدم شما رو کاغذ پیادش کن
به محور مختصات دکارتی بکش چهار تا نقطه مفروض و دلخواه در نظر بگیر مختصات اونارو الان میدونی چیه
این چهار نقطه رو به هم وصل کن یه چهار ضلعی با چهار ضلع بدست میاد که این همون چهار ضلعی وسطی هست
بعد فاصله این چهار نقطه رو از هم پیدا کن (با فرمول رادیکال ایکس یک بعلاوه ایکس دو به توان دو بعلاوه ایگریگ یک بعلاوه ایگریگ دو به توان دو)
که اینا در حقیقت طول اضلاع مربع ها رو به شما میدن اینارو به ترتیب ال یک ال دو ال یه و ال چهار بنامید
حالا شما می تونی طول مراکز هر چهار مربع رو داشته باشی مثلا مربعی که ضلعش ال یک هست مرکزش رادیکال دو دوم ال یک هست
حالا از هر مربع یه نقطه داری و یه طول نیم قطر که یه سرش مراکز مربع هان میتونی با همون فرمول فاصله مراکز رو بدست بیاری
حالا که مختصات نقاط مراکز مربع ها بدست اومد می تونی معادله خط بنویسی
طولشونو حساب کنی ببینی برابرن
برای این که نشون بدی عمودن باید نشون بدی ضرب داخلی دو خط (یا دو بردار صفره) برای این کار می تونی وسط این خطها یه دستگاه مختصات فرض کنی و مختصات مراکز رو تو دستگاه جدید که حاصل انتقال از دستگاه قبل هستند رو بدست بیاری و ضرب داخلی بکنی که حاصل برابر صفر می شه یعنی این دو خط به هم عمودن.
موفق باشید
اگر ابهامی بود اعلام کنید

azarbara
29th November 2010, 04:32 PM
مونده یکی از سوالا که رو اون
فکر می کنم
باور بفرمایید فرصت نیست جمعه امتحان زبان دارم و یکشنبه باید تمرینارو تحویل بدم

lion45
11th January 2011, 09:23 PM
سلام ممنون میشم اگه به سوالم پاسخ بدین
مجموعه زیر را با خواصش بیابید ؟


H={(A,B)|x1a2=x2(a1-1({

A=ماتریسی 2*2 که سطر اول ان x1 x2 وسطر دوم ان 0 0 است
B=ماتریسی 2*2 که سطر اول ان a1 a2 وسطردوم ان 1 0 است
که a1 نا مساوی 0 است

*مینا*
14th January 2011, 07:26 PM
سلام ، خدا قوت


http://uc-njavan.ir/images/y0wh5twd4ugayjmfvv2.jpg



داشتم امروز درس می خوندم ، به همچین چیزی رسیدم
این از انتگرال گیری به دست اومده ...
بازه ها رو باید به جای X قرار بدیم
یادمه تو کلاس استادمون وقتی همچین چیزاییو حل می کرد ، مثلا می گفتش که اگه n عدد صحیح باشه Sin nX صفر می شه ( البته دقیق یادم نمیاد که این جمله رو می گفت یا نه[negaran] )
می شه درستش رو و کاملش رو شما بگین [nishkhand]

Only Math
15th January 2011, 03:09 PM
سلام ، خدا قوت


http://uc-njavan.ir/images/y0wh5twd4ugayjmfvv2.jpg



داشتم امروز درس می خوندم ، به همچین چیزی رسیدم
این از انتگرال گیری به دست اومده ...
بازه ها رو باید به جای X قرار بدیم
یادمه تو کلاس استادمون وقتی همچین چیزاییو حل می کرد ، مثلا می گفتش که اگه n عدد صحیح باشه Sin nX صفر می شه ( البته دقیق یادم نمیاد که این جمله رو می گفت یا نه[negaran] )
می شه درستش رو و کاملش رو شما بگین [nishkhand]
سلام
انتگرال انتگرال معين بوده كه حاصلش رو بعد از انتگرال گرفتن به اين شكل نشون ميدن براي بدست اوردن جوابش بايد عدد بزرگتر رو كه بالا مينويسن بذاري جاي متغير (اينجا x ) بعد عدد كوچيكه كه پايين مينويسن هم بذاري جاي متغير و بعد از هم كم كني.
اگر در ادامه براي n و a هم تعريفي داشته باشي اون رو تو حاصل تفريق قرار ميدي. الزاما هم اگر n عدد صحيح باشه Sin nX صفر نميشه بايد عدد رو بذاري تا مشخص بشه .
اگر مسئله ادامه داره بگو كه بقيه رو هم بگم.[golrooz]

azarbara
15th January 2011, 07:13 PM
سلام ، خدا قوت





http://uc-njavan.ir/images/y0wh5twd4ugayjmfvv2.jpg



داشتم امروز درس می خوندم ، به همچین چیزی رسیدم
این از انتگرال گیری به دست اومده ...

بازه ها رو باید به جای X قرار بدیم
یادمه تو کلاس استادمون وقتی همچین چیزاییو حل می کرد ، مثلا می گفتش که اگه n عدد صحیح باشه Sin nX صفر می شه ( البته دقیق یادم نمیاد که این جمله رو می گفت یا نه[negaran] )
می شه درستش رو و کاملش رو شما بگین [nishkhand]
با تشکر از only math عزیز اگه مراحل بالا رو انجام بدیم داریم
http://www.codecogs.com/eq.latex?\frac{a}{2n\pi }(sin\frac{4na\pi }{3a}-sin\frac{2an\pi}{3a})= \frac{a}{2n\pi }(sin\frac{4n\pi }{3}-sin\frac{2n\pi}{3})

حال اگر از تبدیل حاصلجمع به حاصلضرب برای داخل پرانتز استفاده کنیم (http://www.codecogs.com/eq.latex?sin\alpha -sin\beta =2cos\frac{\alpha +\beta }{2}sin\frac{\alpha -\beta }{2} ) عبارت بالا برابر عبارت زیر خواهد شد
http://www.codecogs.com/eq.latex?2cos\frac{n\pi }{2}sin\frac{n\pi }{3} و چون http://www.codecogs.com/eq.latex?cos\frac{n\pi }{2}=0 پس جواب عبارت بالا همواره برابر صفر خواهد شد

*مینا*
15th January 2011, 07:38 PM
خیلی خیلی خیلی ممنون از پاسخ گوییتون [golrooz][golrooz]

فقط یه مسئله ای که هست ، اینه که ما تو امتحانمون زمان کافی برای حساب کردن جواب آخر نداریم ...
راستش سوالم رو شاید اون طور که باید نپرسیدم ، منظورم از سوالم این بود که ، روشی که بدون محاسبه کردن بفهمم جواب انتگرالم می شه صفر ( و اگه صفر نمی شه باید حلش کنم ... ) رو می خواستم

مثلا در Sin nX اگهn عدد صحیح ، نه کسری ، باشه جواب صفر میاد [nishkhand]

الان تو مثالی که من گذاشتم هم شما جوابش رو صفر آوردین ، فک کنم جمله ی پر رنگ شده ی بالا درست باشه دیگه ؟ نه ؟ [tafakor]

-------------------
بعد یه سوال دیگه این فرمول ها رو با چه نرم افزاری نوشتین [soal]

http://www.codecogs.com/eq.latex?%5Cfrac%7Ba%7D%7B2n%5Cpi%20%7D%28sin%5Cfr ac%7B4na%5Cpi%20%7D%7B3a%7D-sin%5Cfrac%7B2an%5Cpi%7D%7B3a%7D%29=%20%5Cfrac%7Ba %7D%7B2n%5Cpi%20%7D%28sin%5Cfrac%7B4n%5Cpi%20%7D%7 B3%7D-sin%5Cfrac%7B2n%5Cpi%7D%7B3%7D%29

Only Math
15th January 2011, 08:59 PM
خیلی خیلی خیلی ممنون از پاسخ گوییتون [golrooz][golrooz]

فقط یه مسئله ای که هست ، اینه که ما تو امتحانمون زمان کافی برای حساب کردن جواب آخر نداریم ...
راستش سوالم رو شاید اون طور که باید نپرسیدم ، منظورم از سوالم این بود که ، روشی که بدون محاسبه کردن بفهمم جواب انتگرالم می شه صفر ( و اگه صفر نمی شه باید حلش کنم ... ) رو می خواستم

مثلا در Sin nX اگهn عدد صحیح ، نه کسری ، باشه جواب صفر میاد [nishkhand]

الان تو مثالی که من گذاشتم هم شما جوابش رو صفر آوردین ، فک کنم جمله ی پر رنگ شده ی بالا درست باشه دیگه ؟ نه ؟ [tafakor]

-------------------
بعد یه سوال دیگه این فرمول ها رو با چه نرم افزاری نوشتین [soal]

http://www.codecogs.com/eq.latex?%5Cfrac%7Ba%7D%7B2n%5Cpi%20%7D%28sin%5Cfr ac%7B4na%5Cpi%20%7D%7B3a%7D-sin%5Cfrac%7B2an%5Cpi%7D%7B3a%7D%29=%20%5Cfrac%7Ba %7D%7B2n%5Cpi%20%7D%28sin%5Cfrac%7B4n%5Cpi%20%7D%7 B3%7D-sin%5Cfrac%7B2n%5Cpi%7D%7B3%7D%29

درمورد راه حل هاي كوتاه خيلي اطلاعات ندارم ضمن اينكه اكثرا اين راه حلها براي امتحانات قابل قبول نيستن.
براي نوشتن فرمول هاي رياضي نرم افزار هاي متنوعي وجود داره. اگه فقط براي تايپ ميخواي mathtype هم خوبه هم كار باهاش راحته ضمنا هم به صورت جداگانه كار ميكنه و هم جز منو word و powerpoint ميشه.
اما اگر ميخواي محاسبات هم انجام بده هم matlab و هم maple خوبه كه كار با maple راحتتره. (احتمالا برنامه هاي ديگه هم هست كه من نميشناسم.)

azarbara
16th January 2011, 12:31 PM
خیلی خیلی خیلی ممنون از پاسخ گوییتون [golrooz][golrooz]

فقط یه مسئله ای که هست ، اینه که ما تو امتحانمون زمان کافی برای حساب کردن جواب آخر نداریم ...
راستش سوالم رو شاید اون طور که باید نپرسیدم ، منظورم از سوالم این بود که ، روشی که بدون محاسبه کردن بفهمم جواب انتگرالم می شه صفر ( و اگه صفر نمی شه باید حلش کنم ... ) رو می خواستم

مثلا در Sin nX اگهn عدد صحیح ، نه کسری ، باشه جواب صفر میاد [nishkhand]

الان تو مثالی که من گذاشتم هم شما جوابش رو صفر آوردین ، فک کنم جمله ی پر رنگ شده ی بالا درست باشه دیگه ؟ نه ؟ [tafakor]

-------------------
بعد یه سوال دیگه این فرمول ها رو با چه نرم افزاری نوشتین [soal]

http://www.codecogs.com/eq.latex?%5Cfrac%7Ba%7D%7B2n%5Cpi%20%7D%28sin%5Cfr ac%7B4na%5Cpi%20%7D%7B3a%7D-sin%5Cfrac%7B2an%5Cpi%7D%7B3a%7D%29=%20%5Cfrac%7Ba %7D%7B2n%5Cpi%20%7D%28sin%5Cfrac%7B4n%5Cpi%20%7D%7 B3%7D-sin%5Cfrac%7B2n%5Cpi%7D%7B3%7D%29

سلام خدمت شما مینا خانم
اونی که پر رنگ نوشتین درست نیست درستش اینه http://www.codecogs.com/eq.latex?sin(n\pi ) اگر n صحیح باشه صفر میشه
علتش هم اینه که اگه شما نمودار سینوس رو رسم بکنید ملاحظه می کنید که در ضرایب صحیح عدد پی نمودار محور x رو قطع می کنه پس حاصل http://www.codecogs.com/eq.latex?sin(n\pi ) برابر صفر خواهد شد (اگر n صحیح باشه)
برای نوشتن فرمولا هم به قول معروف کلک رشتی زدم[khande] شما تو این (http://www.forum.mathsoft.ir) سایت عضو بشید تو قمست نوشتن نظر امکان تایپ فرمول های ریاضی هست من اونجا نوشتم و کپی کردم و اینجا پیست کردم

*مینا*
16th January 2011, 07:33 PM
سلام خدمت شما مینا خانم
اونی که پر رنگ نوشتین درست نیست درستش اینه http://www.codecogs.com/eq.latex?sin%28n%5Cpi%20%29 اگر n صحیح باشه صفر میشه
علتش هم اینه که اگه شما نمودار سینوس رو رسم بکنید ملاحظه می کنید که در ضرایب صحیح عدد پی نمودار محور x رو قطع می کنه پس حاصل http://www.codecogs.com/eq.latex?sin%28n%5Cpi%20%29 برابر صفر خواهد شد (اگر n صحیح باشه)
برای نوشتن فرمولا هم به قول معروف کلک رشتی زدم[khande] شما تو این (http://www.njavan.com/forum/redirector.php?url=http%3A%2F%2Fwww.forum.mathsoft .ir) سایت عضو بشید تو قمست نوشتن نظر امکان تایپ فرمول های ریاضی هست من اونجا نوشتم و کپی کردم و اینجا پیست کردم
wow بله درسته :)>-
[nishkhand] روش خوبیه ممنون

ممنون از هر دو [golrooz][golrooz][golrooz][golrooz]

sadeghol
19th February 2011, 04:08 AM
حل مسائله یP = NP میخوام[khabalood]

azarbara
20th February 2011, 05:02 PM
سوال کاملا مبهم هست لطفا واضح تر منظورتان را بیان کنید!!

*alien*
17th March 2011, 01:36 PM
سلام میدونم اینجا جاش نیست ولی یه سئوال ریاضی دارم
اگر محیط مثلثی 60 باشد وارتفاع وارد بر وتر آن12 باشد مساحت آن چند است
همکار تالار ریاضی منتظرم

jupiter_895
6th April 2011, 01:38 PM
با سلام و خسته باشید
دو تا سوال داشتم که میخواستم اگه لطف کنید جواب بدید

مسئله اول: پیدا کردن جواب عمومی :


http://latex.codecogs.com/gif.latex?x%5Csqrt%7B1+y%5E2%7D+y%5Cfrac%7Bdy%7D%7 Bdx%7D%5Csqrt%7B1+x%5E2%7D=0

مسئله دوم: پیدا کردن جواب خصوصی :


http://latex.codecogs.com/gif.latex?%281+e%5Ex%29y%5Cfrac%7Bdy%7D%7Bdx%7D=e% 5Ey&space;;y%280%29=0

در ضمن e به توان جه عددی 0 میشه؟

azarbara
7th April 2011, 02:40 AM
دوست عزیز دو سوال اول شما معادله دیفرانسیل تفکیک پذیر هست که برای سوال اول با کمی تغییرات داریم

http://www.codecogs.com/eq.latex?%5Cfrac%7Bx%7D%7B%5Csqrt%7B1+%7Bx%7D%5E%7 B2%7D%7D%7Ddx=-%5Cfrac%7By%7D%7B%5Csqrt%7B1+%7By%7D%5E%7B2%7D%7D% 7Ddy

که هر دوی این انتگرال ها با یک تغییرمتغیر ساده ( عبارات داخل پرانتز را برابر u بگیرید حل می شوند)
و اما معادله دوم که با کمی تغییرات بصورت زیر در می آید
http://www.codecogs.com/eq.latex?y%7Be%7D%5E%7B-y%7Ddy=%5Cfrac%7Bdx%7D%7B1+%7Be%7D%5E%7Bx%7D%7D
انتگرال گیری از این معادلات به سادگی معادله اول نیست سمت چپ با دو بار جز به جز بصورت http://www.codecogs.com/eq.latex?-y%7Be%7D%5E%7B-y%7D-%7Be%7D%5E%7B-y%7D برای حل انتگرال طرف دوم من از تغییر متغیری به ظاهر عجیب استفاده کردم (به یاد دارم دوران کارشناسی از این کارها خیلی انجام میدادم)



http://www.codecogs.com/eq.latex?%7Be%7D%5E%7Bx%7D=%7Btg%7D%5E%7B2%7Dt%5CR ightarrow%20x=ln%7Btg%7D%5E%7B2%7D=2lntgt


حال اگر مشتق بگیریم داریم


http://www.codecogs.com/eq.latex?dx=2%5Cfrac%7B%7Bsec%7D%5E%7B2%7Dt%7D%7Bt gt%7Ddt=%5Cfrac%7Bdt%7D%7Bsintcost%7D

با جایگذاری مقادیر در عبارت اصلی خواهیم داشت

http://www.codecogs.com/eq.latex?%5Cint%20%5Cfrac%7Bdt%7D%7B%7Bsec%7D%5E%7 B2%7Dtsintcost%7D=%5Cint%20cotgt%20dt
که جواب معادله اخیر نیز براحتی قابل محاسبه است
و اما جواب سوال سوم منفی بینهایت است
موفق باشید

jupiter_895
7th April 2011, 03:39 PM
خیلی ممنون از راهنماییتون، میشه قسمت تغیر متغیر و جزء به جزو یکم بیشتر توضیح بدین؟

azarbara
7th April 2011, 05:39 PM
برای جزء به جزء از تغییر متغیر http://www.codecogs.com/eq.latex?y=u و http://www.codecogs.com/eq.latex?%7Be%7D%5E%7B-y%7Ddy=dv استفاده می کنیم
اگر از اولی مشتق و از دومی انتگرال بگیریم داریم http://www.codecogs.com/eq.latex?dy=du و http://www.codecogs.com/eq.latex?-%7Be%7D%5E%7B-y%7D=v و اگر از فرمول مربوط به انتگرال گیری جزء به جزء (http://www.codecogs.com/eq.latex?uv-%5Cint%20vdu)استفاده کنیم داریم

http://www.codecogs.com/eq.latex?-y%7Be%7D%5E%7B-y%7D+%5Cint%20%7Be%7D%5E%7B-y%7Ddy=-y%7Be%7D%5E%7B-y%7D-%7Be%7D%5E%7B-y%7D

azarbara
7th April 2011, 05:54 PM
این گونه تغییر متغیر ها معمولا جایی مطرح نمی شود و بستگی به خلاقیت شما دارد
اگر قسمت تغییر متغیر برای شما مبهم است می توانید از تغییرمتغیر زیر استفاده کنید که بی شک همه از آن استفاده می کنندhttp://www.codecogs.com/eq.latex?1+%7Be%7D%5E%7Bx%7D=t%5CRightarrow%20x=ln %28t-1%29%5CRightarrow%20dx=%5Cfrac%7Bdt%7D%7Bt-1%7D
و با جایگذاری خواهیم داشت


http://www.codecogs.com/eq.latex?%5Cint%20%5Cfrac%7Bdt%7D%7Bt%28t-1%29%7D

که با تجزیه کسر خواهیم داشت


http://www.codecogs.com/eq.latex?%5Cint%20%5Cfrac%7Bdt%7D%7Bt%28t-1%29%7D=-%5Cint%20%5Cfrac%7Bdt%7D%7Bt%7D+%5Cint%20%5Cfrac%7 Bdt%7D%7Bt-1%7D=-lnt+ln%28t+1%29=ln%5Cfrac%7Bt+1%7D%7Bt%7D

درباره انتگرال گیری جزء به جزء من از جدول استفاده کردم اگر شما استفاده از جدول رو برای حل انتگرال جزء به جزء بلد نیستید اعلام کنید تا بهتون بگم چطوری می تونید راحت تر انتگرال های جزء به جزء رو حل کنید

jupiter_895
7th April 2011, 11:50 PM
دستتون درد نکنه،متوجه شدم.
خدا خیرتون بده من تازه دارم این روشهارو یاد میگیرم.

ormazda
14th April 2011, 12:29 PM
با درود
دوستان عزیز کسی میتونه من رو در این مورد راهنمایی کند ؟ و بگوید منظور از تعبیر هندسی چیست .

با احترام

azarbara
15th April 2011, 05:08 PM
درود بر شما
تعبیر هندسی یک مساله به زبان ساده رسم شکل مربوط به آن مساله هست
اگر جای خاصی این مطلب رو دیدید اون رو همین جا قرار بدید تا به طور کامل پاسخ داده بشه

ormazda
16th April 2011, 05:30 PM
با درود
azarbara عزیز در کتاب "ریاضیات چیست " در بخش حد در توضیح این جمله که "اگر از ابتدای دنباله 2،1/3،1/1 ،..... به قدر کافی دور شویم ، می توانیم امیدوار باشیم که اختلاف هریک از جمله های دنباله با صفر به قدر دلخواه کوچک باشد" منظور از "به قدر کافی دور" چقدر دور و " به قدر دلخواه کوچک " چقدر کوچک است ؟ گفته بود که تعبیر هندسی به روشن تر شدن موضوع کمک خواهد کرد.
منظور من هم تعبیر هندسی در این مورد است.


با احترام

azarbara
18th April 2011, 08:53 AM
سلام
دوست عزیز، من واقعا منظور شما رو از این دنباله متوجه نشدم در صورت امکان رابطه کلی دنباله رو بنویسید دنباله شما از یک http://njavan.com/forum/data:image/png;base64,iVBORw0KGgoAAAANSUhEUgAAAB4AAAA2CAIAAAB frb6uAAAAcUlEQVRYhe3WsQ2AIBCF4TcW+7DI9VgzBJVM4BDgC q4Adm5gDOYI6Ev++svlqodST6VAmvQ7+nBGYJMabbas8ZC8eMC 7fS66xACIjaRJ96dbI016IDpZCB4V1nGuJk36BzTnZEd60h1C+ hN0a6RJ33cBLUbv+gRfavkAAAAASUVORK5CYII=شروع می شود جمله دوم یک سوم است و جمله سوم 2 و ...
من نتونستم بین این ها رابطه ای پیدا کنم
ولی من این جا دنباله ای می نویسم و سوال شما رو توضیح میدم
دنباله http://www.codecogs.com/eq.latex?%5Cfrac%7B1%7D%7Bn%7D http://njavan.com/forum/data:image/png;base64,iVBORw0KGgoAAAANSUhEUgAAAB4AAAA2CAIAAAB frb6uAAAApUlEQVRYhe3WsQ2EMAwF0D9W9vEi6aFmiFRhAoYIr MAKcN0Vp2tjcGQE6Eu/frKs2DG2/eMUkCbdRq9diJDiRodp9mjI3A/A0C3PorcxAVFG0qSvp60hTfpGdBFEHErK96maNOkTtPkIUWn7V fa+Xmf5LQpjo5Wqs+g7yEbbj3aVLoIY+tWBXqZgumx0umVYFDq LfVjqdNMc1un/D+nzQlpDmnQ9X0a37CYpmQspAAAAAElFTkSuQmCC را در نظر بگیرید که در آن http://njavan.com/forum/data:image/png;base64,iVBORw0KGgoAAAANSUhEUgAAADcAAAAYCAIAAAD RQ/LyAAAA10lEQVRIie2VwQ3DIAxF/1jswyK+w5khciITZIiQFVih7a2Hpi0CKdiJVKiE9I+An76/DW73R/9Cc4JBOShZlME6wJmtPdwBZTSKoJbQmqziZYcSUHpNysb2lF4T QAehDNYBBEy+rZdeMyDmCSCA9Cyu9zZCdjejjEYR9Mq7vGoQwD//aYUYFGVhYfiiUcSLQfq4yI6MclvUqT7uPlX2V0HJ3nfIK4n3Od/Lbyhf4jetmHHBPhfnMgUV5SqlZHfhwoyfU0pZ92Yf0p//n3/493SrQTkoe9QT3ZNhmvp29igAAAAASUVORK5CYII= http://www.codecogs.com/eq.latex?n%5Cgeq%201 در این صورت جملات این دنباله بصورت زیر می باشد:
http://www.codecogs.com/eq.latex?%7B1,%5Cfrac%7B1%7D%7B2%7D,%5Cfrac%7B1%7D %7B3%7D,...,%5Cfrac%7B1%7D%7B100%7D,...,%5Cfrac%7B 1%7D%7Bn%7D,...%7D
در این دنباله ملاحظه می شود که اختلاف اولین جمله دنباله از صفر برابر 1 اختلاف دومین جمله یک دوم http://njavan.com/forum/data:image/png;base64,iVBORw0KGgoAAAANSUhEUgAAAB4AAAA2CAIAAAB frb6uAAAAnElEQVRYhe3Wuw2AIAAE0BuLfViEXmqGoJIJGEJcw RXUzsKYaEMEwaC55OqXyyV8MC9rpYA06Wf01AkFOVSjhQ81Bgn aAKYbv0XPzgJKOtKk36dzQ5p0Q/QgoXArtm+nNWnSv6YvH8mMP1SsdS9Ppzn9hYzRl6ajF1BCT4W2 dhYHF7RBcfqcYoMcrf1+zSZVThgkI6QboZ+FNOl4NvtNzbBNNa GmAAAAAElFTkSuQmCCاست که نصف اختلاف اول یعنی1 می باشد پس به صفر نزدیکتر شد و هر چه شما پیش بروید مثلا جمله دهم را در نظر بگیرید اختلاف برابر یک دهم http://njavan.com/forum/data:image/png;base64,iVBORw0KGgoAAAANSUhEUgAAACkAAAA2CAIAAAD VgifvAAAAtklEQVRYhe3XsQ3DIBAF0D8W+7AIvakZwpU9QYaAr OAVsDsXTiJLkaIcPptEfOn3D46TjkOel1oBbdq03zJ1xsHGera 5pSo1Tz4Aobu3ZuexB5wdadOmfZKtF9q0aV9tRwsHnfTDH92bN m3azdiay1+RrbT8iWuuuIDp24N9DA/p5/WLXvv4KU4+7GeXiD9oRwtn/JTnpaArtW1JVx6t+euxtzzPcYm950Wwjl2cH7dPC23aLdgrl1p gMBTfYeYAAAAASUVORK5CYII= است که نسبت به اعداد قبلی به صفر نزدیکتر است حال اگر برای مثال جمله پانصدم را در نظر بگیرید اختلاف برابر http://njavan.com/forum/data:image/png;base64,iVBORw0KGgoAAAANSUhEUgAAADQAAAA2CAIAAAA H0i23AAABBUlEQVRoge3YzRGDIBAF4C2LfrYR7nKmCE5agUWAL dhC4i0HxYkQwkhGJc6beSdnlU8Ff5Yez6na0OUC4ICrKDfHjY2 QxLZinOhdlbjJKU2kmwG43ekMkeQOOOCAA67KAAcccMAdgrNMk s6LaW9z5SoKcMABB9wpOKe0UGOdOP82O6aP9Btu6MX6HjymlVS Ms+xNTumUr+VFn/oByxYU4MZGbL8aOhPf3wVNyeGzBSW4+aDBsaKNlkn65fKxxZkt KMGl+pXB9mjscJdsQQnOcmKFBq3CdT7NiR862YJSnJ9wb6B5Im 76mOvwqYGzBXtx4RlvctYzJbtaL5N9x10f4IAD7l9wLye9hYey 8Uf8AAAAAElFTkSuQmCC یک پانصدم خواهد شد که نسبت به اعداد قبلی به صفر خیلی نزدیک تر است.
منظور از این که این اختلاف به اندازه دلخواه به صفر نزدیک باشد این است که عددی نزدیک صفر مد نظر شما هست و می خواهید که اختلاف، کمتر از این عدد باشد شما می توانید مخرج کسر http://njavan.com/forum/data:image/png;base64,iVBORw0KGgoAAAANSUhEUgAAAB4AAAA2CAIAAAB frb6uAAAAvklEQVRYhe3XsQ0DIQwFUI/FPl7EPdQMQQUTMATcCrfCJV2K6CJAOhQOiIhk6RfQPCELgQ3H4 zkpwDTTffQuBQGGabTwcUZBotIAWm7/RR/OABA6ppn+PX03TDO9EB0QCL6KseucmmmmmV6Mjkp/HopzTULtY04dlQbhJb7F5m64RluktE2wOIwOCOk80DweXNObF2 lnU2y7aGeyjr3Y9tAWs3+ktdAVuqjsnWHpig6Y3eLzwxx1+TrD NNP1vAC2dM3wiu3O4wAAAABJRU5ErkJggg== را آنقدر بزرگ فرض کنید (معنای به انذازه کافی دور شدن از جمله اول) که حاصل این کسر همان عددی باشد که در نظر گرفته اید
برای روشن تر شدن یک مثال عددی می زنم مثلا عدد http://njavan.com/forum/data:image/png;base64,iVBORw0KGgoAAAANSUhEUgAAAIgAAAAYCAIAAAC uviJGAAABAklEQVRoge2YsQ2DMBBFbyzvwyLu7dpDpAoTMATOC qxgu0uBoqRA+C464JR86bX/f9BDWDKV2oBB6PInAJtAjFEgxigQYxSIMQrEGAVijmYJztMwS4 MQczRLcJ7clIVBiDmcHBNRCg9Zak/MffBEnsgPo6Cxm+LUGilRWVEWk2Na90RuuilOrZESlZVSWxlv0 o97R8w8kHdxKbVJjq9uilNrpERl5QQxrOOrm+LUGilRWdEX8/51rrzmO3RTnFojJSor+mI+V5lWmClOrZESlRV9MZs48kRJmvpr zhETHMQIOUdMqa2M0/Vv++uIxeSYvrj5AVJwJWMUiDEKxBgFYozyBIcfBkJgh0QcAAAA AElFTkSuQmCC http://www.codecogs.com/eq.latex?0.0000001 که خیلی به صفر نزدیک است را در نظر می گیریم و می خواهیم عددی را در دنباله برای n درنظر بگیریم که اختلاف این عدد با صفر برابر http://www.codecogs.com/eq.latex?0.0000001 گردد لذا می توانیم در کسر http://njavan.com/forum/data:image/png;base64,iVBORw0KGgoAAAANSUhEUgAAAB4AAAA2CAIAAAB frb6uAAAAvklEQVRYhe3XsQ0DIQwFUI/FPl7EPdQMQQUTMATcCrfCJV2K6CJAOhQOiIhk6RfQPCELgQ3H4 zkpwDTTffQuBQGGabTwcUZBotIAWm7/RR/OABA6ppn+PX03TDO9EB0QCL6KseucmmmmmV6Mjkp/HopzTULtY04dlQbhJb7F5m64RluktE2wOIwOCOk80DweXNObF2 lnU2y7aGeyjr3Y9tAWs3+ktdAVuqjsnWHpig6Y3eLzwxx1+TrD NNP1vAC2dM3wiu3O4wAAAABJRU5ErkJggg== n را برابرhttp://www.codecogs.com/eq.latex?n=%5Cfrac%7B1%7D%7B10000000%7D http://njavan.com/forum/data:image/png;base64,iVBORw0KGgoAAAANSUhEUgAAAJ0AAAAYCAIAAAB vOWjqAAABSElEQVRoge2YwRGDIBBFtyz6oRHveKYIT1qBRYgt2 EKSWw46aoiyEIgw6868Q5j5syt544rC4/li6AHZr4D5B+yVJuyVJuyVJuyVJuyVJld6nZSoQA7Z93wHLvcq epN7z3fgwKupNUAF0LTb70rUU3wzU2sArcb82ybP8f1qag2iV3 LWeXifTUpUs3IL2Z02Q722Ei+CZuID5RTx6RLgdS631mplovnZ NY5LXGeDYydoJj5QThGfLkFeBwn7A066847L6yC3aX/WEc3EB8op4tMlyOvYi/2/by0v83owIdBMfKCcIj5dgrx2zcdT0Fou/PJ8dc/h9VniOKmhmfhAOUV8ugR4beVyGN6WqV5OnF73O3HsAc3EB8op4 tPF06s1ypO+dGJemVR8e92P9WUJcMV5mEkIfx+mCXulCXulCXu lCXulCXulyRt3AFhAzZVH9gAAAABJRU5ErkJggg==بگیر م می بینم که به اندازه کافی از جمله اول دور شده ایم و حاصل کسر به انذازه دلخواه به صفر نزدیک شده است
موفق باشیدhttp://njavan.com/forum/data:image/png;base64,iVBORw0KGgoAAAANSUhEUgAAABAAAAAYCAIAAAB 8wupbAAAAiElEQVQ4je2SvQ2AIBBGv7Fun1vkeqgZwgonYAhwB VZQOws1ol4wFiaamFDwEh4/L6AfxlsDv/BWIVkHCND4bS5kc+2EZB0oGJ7XZUMCCqkieBZAuC2wKkSGgOOK 2dCGmtAFKrY/oCa0DeBMp6MieF4SnR+gCrsbHxKpQuSi+hIAV1kf/hqfEyaXAhBoBVVzyQAAAABJRU5ErkJggg==

ormazda
18th April 2011, 03:03 PM
با درود و سپاس از azarbara عزیز

از این سوال منظور و خواسته من تعریف حد با استفاده از تعبیر هندسی است و اینکه تعبیر هندسی چه مفهومی به طور کلی دارد و چگونه به روشن تر شدن موضوع در این تعریف کمک می کند ، تعریف مزبور به طور کامل در زیر می اید:

"دنباله ای که جمله n ام آن http://ormazd1.persiangig.com/image/2.bmp است:
http://ormazd1.persiangig.com/image/3.bmp (1)
حدش وقتی n به سمتی بینهایت می رود ، صفر است:
http://ormazd1.persiangig.com/image/5.bmp وقتی http://ormazd1.persiangig.com/image/4.bmp (2)
در اینجا سعی می کنیم بگوییم که منظور از این دقیقا چیست. در این دنباله هر چه جلوتر می رویم، جمله ها کوچک و کوچک تر می شوند. پس از جمله 100ام همه جمله ها کوچکتر از 1/100 اند، پس از جمله 1000 ام همه جمله ها کوچکتر از 1/1000 اند، وبه همین ترتیب هیچ یک از جمله ها واقعا صفرنیست.ولی اگر از ابتدای دنباله (1) به قدر کافی دورشویم ، می توانیم امیدوار باشیم که اختلاف هر یک از جمله های دنباله با صفر به قدر دلخواه کوچک باشد.تنها مشکلی که این توضیح دارد ، این است که معنای عبارتهایی که در سطور بالا با حروف خمیده و پررنگ چاپ شده کاملا واضح نیست . "به قدر کافی دور" چقدر دور است و "به قدر کافی کوچک" چقدر کوچک است؟ اگر بتوانیم معای دقیقی برای این عبارتها قائل شویم می توانیم معنای دقیقی به رابطه حدی (2) بدهیم. تعبیر هندسی به روشن شدن وضع کمک خواهد کرد.اگرجمله های دنباله (1) را به صورت نقطه های متناظرشان روی محور اعداد نشان دهیم، ملاحظه می کنیم که جمله های دنباله در حوالی نقطه 0 مجتمع می شوند.

azarbara
20th April 2011, 08:18 PM
من ببینم می تونم یه فایل فلش از نحوه چگونگی این کار درست کنم و به شما نشون بدم تا به طور کامل معنی کلمات پررنگ و خمیده رو به عینه ببیند
من سعی می کنم این کارو انجام بدم
اگر هم نتونستم شرمنده

saeed052
29th April 2011, 06:30 PM
سلام اين دو معادله رو در نظر بگيريد
acosx-bcosy=A
asinx-bsiny=B
که aوbوAوBعدد هستند.من مي خوامxوy رو به روش معادلات کلاسيک نوع اول به دست بيارم

اگه کمکم کنيد ممنون ميشم

azarbara
30th April 2011, 02:13 PM
لطفا منظورتان را از روش معادلات کلاسیک نوع اول بیشتر توضیح دهید تا شاید بتوان کاری برای شما انجام داد

Jopiter
27th May 2011, 06:34 PM
با درود
معنی این واژه چیست؟
coplanar

ستاره کویر
27th May 2011, 06:50 PM
تنها چیزی که پیدا کردم ...[tafakor] کنجکاو شدم بدونم!!![soal]

شيمى : هم صفحه

http://www.l3go.bugge.com/articles/miscellaneous/Reviewing_tutorial/Reviewing-Non-coplanar.jpg

http://zhlt.info/pics/coplanar-2.gif

azarbara
31st May 2011, 10:51 PM
پاسخ "بی ستاره" عزیر کاملا صحیح است
در فرهنگ ریاضی planar یعنی مسطح و coplanar معنی هم صفحه یا در یک صفحه واقع شدن را دارد

ریباز
19th June 2011, 08:11 AM
[golrooz]سلام دوستان
ایا می توان مجموعه ی از اعداد رو یافت که فضا رو مشخص کنند یعنی متناسب با هر نقطه در فضا بتوان یه عددو نسبت داد(ترکیبی خطی از دو عدد مختلط )ایا این مجموعه معرفی شده لطفا اگه هست سایت مرجع یا کتاب معرفی کنید......[golrooz]

azarbara
19th June 2011, 11:11 AM
با سلام لطفا سوالتان را در بخش سوالات ریاضی در بالای همین صفحه بپرسید
ما فضاهای متفاوت با توپولوژی های متفاوت داریم ولی در فضای اقلیدسی (همان فضایی که به عنوان صفحه اقلیدسی می شناسیم) هر نقطه از فضا را می توان با یک جفت عدد (یک عدد مختلط) نشان داد.
پس اعداد مختلط وسیعترین مجموعه اعداد ما هستند.
بسیار خوب است که شما برای این زمینه ها کتاب هندسه اقلیدسی و نااقلیدسی را مطالعه بفرمایید. تا با فضاهای متفاوت آشنا شوید. همچنین کتاب توپولوژِی عمومی مانکرز نیز مفید خواهد بود.

Maani Parvaz
27th September 2011, 08:03 PM
با سلام خدمت دوستان گرامی
من این ترم درس معادلات دیفرانسیل را در مقطع کارشناسی ناپیوسته دارم اما هنوز از نظر ریاضیات ضعیف هستیم.
خواهشا با ذکر چندین مثال تفاوت معادله سرعت با معادله حرکت را بگید, همچنین من در یک مثال دیدم که مشتق معادله حرکت به معادله سرعت تبدیل شد یا برابر شدند, علتش چیه؟ چرا باید از معادله حرکت مشتق بگیریم؟ واقعا دوستان در این زمینه خیلی ضعیف هستم و دوست دارم کاملا منو راهنمایی کنید, چطوری میشه این مسائل رو به خوبی درک کرد؟

Only Math
27th September 2011, 08:33 PM
این موردی که شما بیان کردید بیشتر مربوط به فیزیک مکانیک هست تا معادله دیفرانسیل!
معادله حرکت معادله مکان (x) هست که اگه ازش مشتق بگیریم به معادله سرعت (v) تبدیل میشه
از کتاب معادلات دیفرانسیل دکتر نیکوکار میتونید برای این واحدتون کمک بگیرید ولی جواب سوال شما مربوط به فیزیک 1 هست

jupiter_895
15th October 2011, 03:47 PM
سلام یه سوال از اعداد مختلط داشتم: فرض کنید http://www.codecogs.com/eq.latex?z%5Cneq%201 و ریشه n ام واحد باشد، مجموع http://www.codecogs.com/eq.latex?%281+2z+3%7Bz%7D%5E%7B2%7D+...+n%7Bz%7D%5 E%7Bn-1%7D%29 را محاسبه کنید؟

من اینطوری حلش کردم:


http://www.codecogs.com/eq.latex?%5Cinline%20%281+2z+3%7Bz%7D%5E%7B2%7D+.. .+n%7Bz%7D%5E%7Bn-1%7D%29=%5Cfrac%7B%281-z%29%281+2z+3%7Bz%7D%5E%7B2%7D+...+n%7Bz%7D%5E%7Bn-1%7D%29%7D%7B%281-z%29%7D=%5Cfrac%7B%281+2z+3%7Bz%7D%5E%7B2%7D+...+n %7Bz%7D%5E%7Bn-1%7D-z-2%7Bz%7D%5E%7B2%7D-3%7Bz%7D%5E%7B3%7D-...-n%7Bz%7D%5E%7Bn%7D%29%7D%7Bz-1%7D



بقیشو بلد نیستم[negaran]

jupiter_895
17th October 2011, 01:11 PM
یعنی هیچکس جوابشو نمیدونه؟؟؟؟؟؟؟؟؟[soal]

466
17th October 2011, 10:06 PM
تعداد افرازها

arad2000_gh
25th October 2011, 02:29 PM
با سلام
اگه امکان داره جواب این جزء صحیصح رو بدین
لطفا با راه حل[golrooz]



[ log arc tan 1390]

azarbara
27th October 2011, 01:00 AM
با سلام
اگه امکان داره جواب این جزء صحیصح رو بدین
لطفا با راه حل[golrooz]



[ log arc tan 1390]


این سوال مربوط به کدام درس است لطفا بیشتر توضیح دهید

arad2000_gh
27th October 2011, 03:08 PM
این سوال مربوط به کدام درس است لطفا بیشتر توضیح دهید
این سوال مربوط به ریاضی عمومی میشه
یا بهتر بگم مربوط به ریاضی پیش دانشگاهی میشه

fantastic girl
27th October 2011, 07:00 PM
برای n بزرگ تر و مساوی 2 وعدد مختلط دلخواه c ثابت کنید مجموع ریشه های n ام c برابر صفر است.
شدید به جوابش نیاز دارم جون هرکی دوست دارید بزارید[golrooz][golrooz]

azarbara
28th October 2011, 10:54 AM
سلام یه سوال از اعداد مختلط داشتم: فرض کنید http://www.codecogs.com/eq.latex?z\neq 1 و ریشه n ام واحد باشد، مجموع http://www.codecogs.com/eq.latex?(1+2z+3{z}^{2}+...+n{z}^{n-1}) را محاسبه کنید؟

با سلام و شرمندگی بخاطر تاخیر بسیار زیادی که در گرفتن پاسخ رخ داده است.
با کمی دقت این مسئله حل می شود

http://www.codecogs.com/eq.latex?\left(1+2z+...+n{z}^{n-1} \right)=\frac{d}{dz}(z+{z}^{2}+...+{z}^{n})=\frac{ d}{dz}(\frac{1-{z}^{n}}{1-z})
دقت کنید که در اینجا در مساوی دوم از ریشه n ام بودن استفاده شده است و جواب سوال شما صفر می شود
سربلند باشید

fantastic girl
30th October 2011, 03:49 PM
خواهش می کنم یکی جواب سوالمو بده [negaran]

ALIREZA_2010
30th October 2011, 08:48 PM
با سلام خدمت دوستان گرامی
من این ترم درس معادلات دیفرانسیل را در مقطع کارشناسی ناپیوسته دارم اما هنوز از نظر ریاضیات ضعیف هستیم.
خواهشا با ذکر چندین مثال تفاوت معادله سرعت با معادله حرکت را بگید, همچنین من در یک مثال دیدم که مشتق معادله حرکت به معادله سرعت تبدیل شد یا برابر شدند, علتش چیه؟ چرا باید از معادله حرکت مشتق بگیریم؟ واقعا دوستان در این زمینه خیلی ضعیف هستم و دوست دارم کاملا منو راهنمایی کنید, چطوری میشه این مسائل رو به خوبی درک کرد؟

سلام. شما روی کیلومتر ماشین نگاه میکنید در قسمت کیلومتر شمار چی می بینید؟ می بینید که نوشته km/h ، که سرعت رو نشون میده. km همون مسافته یعنی حرکت، h هم همون زمانه. که تقسیم اینا شده سرعت متوسط. وقتی مشتق بگیریم میشه سرعت لحظه ای. شما برای مثال یک معادله درجه دوم داشته باشید مثل: x = t^2 + 3t + 4 حالا اگر از این مشتق بگیریم میشه چی؟ مشتق مکان(حرکت) نسبت به زمان میشه v . درنتیجه داریم: v = 2t + 3 . حالا اگر از این دوباره مشتق بگیریم میشه شتاب. مشتق v نسبت به زمان برابر a هست. در نتیجه داریم: a = 2 .
مشتق حرکت نسبت به زمان برابر سرعت لحظه ای است که واحدش میشه متر تقسیم بر ثانیه. که سرعت لحظه ای برابر است با شیب مماس بر نمودار مکان زمان.
مشتق سرعت نسبت به زمان برابر مشتق لحظه ای است که واحدش میشه متر تقسیم بر مجذور ثانیه. که مشتق لحظه ای برابر است با شیب مماس بر نمودار سرعت زمان.


اگر که متوجه نشدین بگید تا اثباتشون رو هم براتون بزارم. [golrooz]

arad2000_gh
19th November 2011, 01:31 PM
سلام من یه سوال در باره دنباله ها دارم. اگه دنباله an به عددی مثل L همگرا باشد آنگاه درباره همگرایی یا واگرایی دنباله a2n چه می توان گفت؟ اگر واگراست علتش چیست؟ اگر همگراست آنگاه به چه عددی بر حسب L همگراست؟ اگه براتون امکان داره زود جوابمو بدین.

azarbara
19th November 2011, 02:36 PM
یک دنباله همگرا به یک عدد است اگرو تنها اگر هر زیر دنباله آن همگرا به آن عدد باشد. در سوال شما هم a2n زیر دنباله an است پس همگرا هست و به خود L نیز همگراست.اگر نقطه مبهمی باقی مانده می توانید به کتاب آنالیز رودین بخش دنباله های عددی مراجعه کنید

LUCKLY
9th December 2011, 10:35 AM
دوستان برای اینکه سرگرم بشید این سوال رو حل کنید و اگه حل کردید فقط اعلام کنید که حل کردید یا نه

سرآخر جوابا رو می ذاریم که همه استفاده کنن


file:///C:/Users/p/AppData/Local/Temp/msohtmlclip1/01/clip_image002.gif

سلام من حلش کردم خیلی جالب بود ممنونم بابت مسئله ی زیباتون

skyzare
26th July 2012, 10:49 PM
با سلام .

اساتید یه سوال .

اگر داشته باشیم :

http://latex.codecogs.com/gif.latex?V%28t%29=i_s%28t%29%5Cstar&space;h%28t%29


http://latex.codecogs.com/gif.latex?V%28t%29 و http://latex.codecogs.com/gif.latex?i_s%28t%29 برای ما معلوم باشه میشه اون http://latex.codecogs.com/gif.latex?h%28t%29 را با محاسبات ریاضی به دست آورد ؟ یه چیزی مثل زیر که i و v معلوم هست و دنبال h هستم .
( اون ستاره همون کانولوشن هست )



http://s3.picofile.com/file/7449757204/conv2.png

zabardast
18th February 2013, 02:01 PM
سلام . جوابش صفر میشه


[ log arc tan 1390

david77
3rd March 2013, 04:46 PM
سلام به دوستان چند تا سوال در مورد مشتق داشتم.
http://8pic.ir/images/v99uyh7llxnrvqefa29.png (http://8pic.ir/)



http://8pic.ir/images/gyqa8b2mc1x6qcxpz3wh.pnghttp://8pic.ir/images/job0zi3nim2en8seotd.png (http://8pic.ir/)

david77
3rd March 2013, 04:56 PM
سلام به دوستان چند سوال در مورد مشتق داشتم.
http://8pic.ir/images/v99uyh7llxnrvqefa29.png (http://8pic.ir/)
http://8pic.ir/images/job0zi3nim2en8seotd.png (http://8pic.ir/)
http://8pic.ir/images/gyqa8b2mc1x6qcxpz3wh.png (http://8pic.ir/)

shiny7
3rd March 2013, 05:41 PM
سلام

فرم دیورژانس در دستگاه مختصات کروی و استوانه ای چطوری می شه؟

همین طورم فرم کرل در دستگاه کروی و استوانه ای؟

david77
24th April 2013, 07:44 AM
سلام به دوستان گرامی

کسی میدونه جواب این انتگرال چی میشه؟

∫(e^y)/(cosy-1)/(e^-cot(y/2))

farshadghasemi
24th April 2013, 03:17 PM
سلام به دوستان گرامی

کسی میدونه جواب این انتگرال چی میشه؟

∫(e^y)/(cosy-1)/(e^-cot(y/2))

انتگرالتون d نداره؟

david77
24th April 2013, 08:40 PM
چرا dy داره حواسم نبود بزارم.

david77
5th May 2013, 08:53 PM
سلام به دوستان

یه سوال داشتم لطفا اگر کسی میتواند جواب دهد:

اعضای شورای دانشجویی یه دانشگاه از 3 نفر دانشجوی سال اول، 4 نفر سال دوم، 5 نفر سال سوم و 2 نفر دانشجوی سال چهارم هستند. اگر بخواهیم یک شورای فرعی 4 نفری که دانشجویان سالهای مختلف در آن باشند انتخاب کنیم به چند طریق امکان پذیر است؟

mahmoodmah
6th May 2013, 11:42 AM
ما 4 گروه دانشجو(سال اول ,سال دوم و...) داریم و قراره یک شورای 4 نفره داشته باشیم
یعنی قراره 4 نفر انتخاب بشند که حتما باید از همه سال های مختلف درش حضور داشته باشند
پس کافی تعداد این گروه های دانشجو رو در هم ضرب کنی فقط همین
3x4x5x2=120

mahmoodmah
6th May 2013, 12:02 PM
سلام به دوستان

یه سوال داشتم لطفا اگر کسی میتواند جواب دهد:

اعضای شورای دانشجویی یه دانشگاه از 3 نفر دانشجوی سال اول، 4 نفر سال دوم، 5 نفر سال سوم و 2 نفر دانشجوی سال چهارم هستند. اگر بخواهیم یک شورای فرعی 4 نفری که دانشجویان سالهای مختلف در آن باشند انتخاب کنیم به چند طریق امکان پذیر است؟



ما 4 گروه دانشجو(سال اول ,سال دوم و...) داریم و قراره یک شورای 4 نفره داشته باشیم
یعنی قراره 4 نفر انتخاب بشند که حتما باید از همه سال های مختلف درش حضور داشته باشند
پس کافی تعداد این گروه های دانشجو رو در هم ضرب کنی فقط همین
در حقیقت فقط ظاهر سوال ترسناکه ولی راه حلش آسونه
3x4x5x2=120

david77
11th May 2013, 02:18 PM
سلام به دوستان گرامی
چند تا سوال دیگه هم داشتم می خواستم بدونم جواباش درسته؟
1- در مسابقه شطرنج از 10 بازیکن، 4 نفر روسی، 3 نفر آمریکایی، 2 نفر انگلیسی و یک نفر برزیلی هستند. اگر نتیجه مسابقه به صورت مرتب کردن ملیت بازیکن ها اعلام شود. چند نتیجه ممکن وجود دارد؟

1- جوابش این میشه؟ !1*!2*!3*!4/!10

2- چند پلاک 8 رقمی اتومبیل را می توان تهیه نمود وقتی دو رقم اول آن از حروف لاتین و 6 رقم بعدی از اعداد باشند (تکرار رقم مجاز نیست)
جواب: 26*25*9*9*8*7*6*5

3- 4 فرد a،b،c،d یک گروه موسیقی مرکب از 4 وسیله موسیقی را تشکیل داده اند. اگر هر کدام بتوانند هر 4 وسیله را بنوازند چند ترتیب متفاوت وجود دارد؟
جواب: 4*!4

مدیر بخش علوم پایه
14th May 2013, 03:36 PM
سلام به دوستان گرامی
چند تا سوال دیگه هم داشتم می خواستم بدونم جواباش درسته؟

1- در مسابقه شطرنج از 10 بازیکن، 4 نفر روسی، 3 نفر آمریکایی، 2 نفر انگلیسی و یک نفر برزیلی هستند. اگر نتیجه مسابقه به صورت مرتب کردن ملیت بازیکن ها اعلام شود. چند نتیجه ممکن وجود دارد؟

جوابش این میشه؟ !1*!2*!3*!4/!10

درست

2- چند پلاک 8 رقمی اتومبیل را می توان تهیه نمود وقتی دو رقم اول آن از حروف لاتین و 6 رقم بعدی از اعداد باشند (تکرار رقم مجاز نیست)
جواب: 26*25*9*9*8*7*6*5

با توجه به اینکه در سوال نگفته 6 رقم به صورت عددی 6 رقمی هستند و تنها به اینکه از اعداد تشکیل شده، اشاره کرده پس نیاز به رعایت قوانین عددنویسی نیست (صفر میتواند به عنوان رقم اول نیز باشد)
26*25* 10*9*8*7*6*5

3- 4 فرد a،b،c،d یک گروه موسیقی مرکب از 4 وسیله موسیقی را تشکیل داده اند. اگر هر کدام بتوانند هر 4 وسیله را بنوازند چند ترتیب متفاوت وجود دارد؟
جواب: 4*!4

نیاز به ضرب 4 نداره! این مسئله دقیقا مشابه این است که 4 نفر بخواهند در 4 صندلی بنشینند پس حاصل همان !4 است

david77
15th May 2013, 09:14 PM
سلام به دوستان

می خواستم ببینم این مسئله ها را درست حل کردم؟
1- تعداد بایت هاییرا بیابید که دقیقا 4 یک داشته باشند؟
جواب:http://upload.tehran98.com/img1/dk6hlei5vbkwoiyy8ge_thumb.png (http://upload.tehran98.com/viewer.php?file=dk6hlei5vbkwoiyy8ge.png)
2- با حروف a،b،c،d،e،f،g چند کلمه 4 حرفی می توان نوشت (تکرار حرف مجاز نیست.)
اینم اگه تونستید برام بنویسید.

juliet
16th May 2013, 06:25 PM
سلام دوستان. کسی می تونه اینو اثبات کنه؟http://www.iran-eng.com/images/smilies/frown.gifsh_dokhtar14ممنون می شمhttp://www.iran-eng.com/images/smilies/frown.gif[golrooz][golrooz]

E(x)=np


Var(x)=npq

david77
23rd May 2013, 11:43 AM
سلام به دوستان گرامی

1- دانشجویی در یک امتحان بایستی به 6 سوال از 10 سوال پاسخ دهد او به چند طریق می تواند سوالها را انتخاب کند.
جواب: http://www.iran-eng.com/attachment.php?attachmentid=151289&d=1368764270

2- فردی 8 دوست دارد که می خواهد 5 نفر آنها را به یک مهمانی دعوت کند چند انتخاب وجود دارد؟
الف) اگر دو نفر از دوستان وی با هم اختلاف داشته باشند و نخواهند در یک گروه شرکت کنند.
جواب:http://www.iran-eng.com/attachment.php?attachmentid=151290&d=1368764500&thumb=1 (http://www.iran-eng.com/attachment.php?attachmentid=151290&d=1368764500)
ب) دو نفر از دوستان وی در صورتی که با هم دعوت شوند در مهمانی شرکت می کنند.
جواب این سوالو نمیدونم اگر کسی بلد هست بنویسه.
3- یک گروه 20 نفری را در نظر بگیرید اگر هر فرد بخواهد با افراد دیگر دست بدهد به چند طریق دست دادن امکان پذیر است؟
!20
میخواستم بدونم جواباش درسته؟ خیلی ممنون میشم.

مدیر بخش علوم پایه
24th May 2013, 01:56 PM
سلام به دوستان

می خواستم ببینم این مسئله ها را درست حل کردم؟ درسته
1- تعداد بایت هاییرا بیابید که دقیقا 4 یک داشته باشند؟
جواب:http://upload.tehran98.com/img1/dk6hlei5vbkwoiyy8ge_thumb.png (http://upload.tehran98.com/viewer.php?file=dk6hlei5vbkwoiyy8ge.png)
2- با حروف a،b،c،d،e،f،g چند کلمه 4 حرفی می توان نوشت (تکرار حرف مجاز نیست.)
اینم اگه تونستید برام بنویسید.

برای حرف اول 7، حرف دوم 6، حرف سوم 5 و حرف چهارم 4 انتخاب داریم یعنی: 4*5*6*7

یا به شکل دیگر میتونیم از فرمول ترتیب استفاده کنیم :


http://www.codecogs.com/eq.latex?%5ccolor%7bblue%7d%7bp%287,4%29=%20%5cfra c%7b7%21%7d%7b%287-4%29%21%7d%7d

مدیر بخش علوم پایه
24th May 2013, 02:01 PM
سلام به دوستان گرامی

1- دانشجویی در یک امتحان بایستی به 6 سوال از 10 سوال پاسخ دهد او به چند طریق می تواند سوالها را انتخاب کند.
جواب:

2- فردی 8 دوست دارد که می خواهد 5 نفر آنها را به یک مهمانی دعوت کند چند انتخاب وجود دارد؟

الف) اگر دو نفر از دوستان وی با هم اختلاف داشته باشند و نخواهند در یک گروه شرکت کنند.
جواب:

ب) دو نفر از دوستان وی در صورتی که با هم دعوت شوند در مهمانی شرکت می کنند.
جواب این سوالو نمیدونم اگر کسی بلد هست بنویسه.

3- یک گروه 20 نفری را در نظر بگیرید اگر هر فرد بخواهد با افراد دیگر دست بدهد به چند طریق دست دادن امکان پذیر است؟
!20
میخواستم بدونم جواباش درسته؟ خیلی ممنون میشم.

پاسخ سوال اول و دومتون دیده نمیشه. برای نوشتن فومول، از فومول نویس سایت استفاده کنید

Only Math
24th May 2013, 02:31 PM
سلام به دوستان گرامی

1- دانشجویی در یک امتحان بایستی به 6 سوال از 10 سوال پاسخ دهد او به چند طریق می تواند سوالها را انتخاب کند.
جواب: http://www.iran-eng.com/attachment.php?attachmentid=151289&d=1368764270

2- فردی 8 دوست دارد که می خواهد 5 نفر آنها را به یک مهمانی دعوت کند چند انتخاب وجود دارد؟
الف) اگر دو نفر از دوستان وی با هم اختلاف داشته باشند و نخواهند در یک گروه شرکت کنند.
جواب:http://www.iran-eng.com/attachment.php?attachmentid=151290&d=1368764500&thumb=1 (http://www.iran-eng.com/attachment.php?attachmentid=151290&d=1368764500)
ب) دو نفر از دوستان وی در صورتی که با هم دعوت شوند در مهمانی شرکت می کنند.
جواب این سوالو نمیدونم اگر کسی بلد هست بنویسه.

3- یک گروه 20 نفری را در نظر بگیرید اگر هر فرد بخواهد با افراد دیگر دست بدهد به چند طریق دست دادن امکان پذیر است؟
!20
میخواستم بدونم جواباش درسته؟ خیلی ممنون میشم.




سوال اول:


ترتیب انتخاب مهم نیست پس میشه :

http://www.codecogs.com/eq.latex?c%2810,6%29=%20%5Cfrac%7B10%21%7D%7B6%21% 2810-6%29%21%7D




سوال دوم:



الف) دقتی در مسئله شرط داریم اول باید وضعیت اون رو مشخص کنیم:

اگر بخوایم حالتهایی رو حساب کنیم که اینها با هم نیستند مراحل زیاد میشه
اول باید حالاتی رو حساب کنیم که 1) هیچکدام نیستند. 2) حالتی که نفر اول هست ولی دومی نیست! 3) نفر دوم هست ولی اولی نیست


راه حل دومی هم وجود داره که اسونتر هست:

کل حالاتی که میشه 5 نفر از 8 نفر رو انتخاب کنیم حساب و ان را از تعدادی که هر دو نفر هستند کم کنیم:




کل:

http://www.codecogs.com/eq.latex?p%288,5%29=%5Cfrac%7B8%21%7D%7B5%21%288-5%29%21%7D


هر دو باشند: برای این حالت دو نفر را انتخاب شده فرض میکنیم و از 6 نفر باقی مانده، 3 نفر را انتخاب میکنیم:





http://www.codecogs.com/eq.latex?p%286,3%29=%5Cfrac%7B6%21%7D%7B3%21%286-3%29%21%7D


جواب مسئله:



http://www.codecogs.com/eq.latex?%5Cfrac%7B8%21%7D%7B5%21%288-5%29%21%7D%20-%20%5Cfrac%7B6%21%7D%7B3%21%286-3%29%21%7D




ب) این قسمت یعنی یا هیچکدام از دو نفر نیستند یا هر دو با هم اند:

هیچکدام نباشند:


http://www.codecogs.com/eq.latex?p%286,5%29=%5Cfrac%7B6%21%7D%7B5%21%286-5%29%21%7D


هردو باشند:


http://www.codecogs.com/eq.latex?p%286,3%29=%5Cfrac%7B6%21%7D%7B3%21%286-3%29%21%7D





سوال سوم:

پاسختون درست نیست!نفر اول با 20 نفر دست نمیده بلکه با 19 نفر دست میده! (با خودش نمیتونه دست بده)
نفر بعدی با 18 نفر
و ......
تا نفر 20ام که نیاز نیست با فرد جدیدی دست بده چون 19 نفر قبل باهاش دست دادن

پس پاسخ برابر میشه با : !19

david77
29th May 2013, 07:44 PM
سلام خدمت دوستان
1- ppp ، qqq،rrr را به چند طریق می توان جایگشت داد به طوری که هیچ سه حرف مشابه کنار هم نباشند.

می خواستم جوابشو اینجا بنویسم ولی نمیدونستم چطوری باید بنویسم. اگه کسی میتونه لطفا راهنماییم کنه.[golrooz] (هم از نظر نوشتن هم از نظر حل مسئله)

david77
5th November 2013, 09:13 AM
سلام به دوستان
این انتگرال چطوری میشه حل کرد؟
http://upload7.ir/images/82888155587307634601_thumb.png (http://upload7.ir/viewer.php?file=82888155587307634601.png)

meha1368
5th November 2013, 08:35 PM
سلام به دوستان
این انتگرال چطوری میشه حل کرد؟
http://upload7.ir/images/82888155587307634601_thumb.png (http://upload7.ir/viewer.php?file=82888155587307634601.png)
سلام . دوست عزیز با روش جزء به جزء حل میشه .
کافیه Arctanx=V و x=du
جوابشم میشه
http://www.codecogs.com/eq.latex?%5Cfrac%7B1%7D%7B2%7D%5Cleft%28%7Bx%7D%5E %7B2%7Darctanx-x+arctanx%20%5Cright%29

david77
7th November 2013, 06:40 PM
سلام
این انتگرال چطوری حل میشه؟
http://upload7.ir/images/71347544660405505269_thumb.png (http://upload7.ir/viewer.php?file=71347544660405505269.png)

meha1368
7th November 2013, 09:49 PM
سلام
http://www.codecogs.com/eq.latex?%5Cint%20%5Cfrac%7B%7Bx%7D%5E%7B2%7D+4%7D %7B%7Bx%7D%5E%7B4%7D-1%7D=%5Cint%20%5Cfrac%7B%7Bx%7D%5E%7B2%7D+1+3%7D%7 B%5Cleft%28%7Bx%7D%5E%7B2%7D-1%20%5Cright%29%5Cleft%28%7Bx%7D%5E%7B2%7D+1%20%5C right%29%7D=%5Cint%20%5Cfrac%7B1%7D%7B%7Bx%7D%5E%7 B2%7D-1%7D+%5Cfrac%7B3%7D%7B%7Bx%7D%5E%7B4%7D-1%7D=-%7Btanh%7D%5E%7B-1%7D%28x%29+%5Cfrac%7B3%7D%7B4%7D%7D%28ln%28x-1%29-ln%28x+1%29-2%7Btan%7D%5E%7B-1%7D%28x%29%29

meha1368
7th November 2013, 09:51 PM
سلام
این انتگرال چطوری حل میشه؟
http://upload7.ir/images/71347544660405505269_thumb.png (http://upload7.ir/viewer.php?file=71347544660405505269.png)

البته اگه تو حل انتگرال
http://www.codecogs.com/eq.latex?%5Cint%20%5Cfrac%7B3%7D%7B%7Bx%7D%5E%7B4% 7D-1%7D
مشکل داشتی اطلاع بده

david77
9th November 2013, 08:02 PM
سلام
http://www.codecogs.com/eq.latex?%5Cint%20%5Cfrac%7B%7Bx%7D%5E%7B2%7D+4%7D %7B%7Bx%7D%5E%7B4%7D-1%7D=%5Cint%20%5Cfrac%7B%7Bx%7D%5E%7B2%7D+1+3%7D%7 B%5Cleft%28%7Bx%7D%5E%7B2%7D-1%20%5Cright%29%5Cleft%28%7Bx%7D%5E%7B2%7D+1%20%5C right%29%7D=%5Cint%20%5Cfrac%7B1%7D%7B%7Bx%7D%5E%7 B2%7D-1%7D+%5Cfrac%7B3%7D%7B%7Bx%7D%5E%7B4%7D-1%7D=-%7Btanh%7D%5E%7B-1%7D%28x%29+%5Cfrac%7B3%7D%7B4%7D%7D%28ln%28x-1%29-ln%28x+1%29-2%7Btan%7D%5E%7B-1%7D%28x%29%29
میشه قسمت سومشو توضیح بدی که چطوری انتگرال دومی تبدیل به قسمت سوم شد؟
اون انتگرالی هم که گفتی مشکل دارم.

meha1368
9th November 2013, 11:39 PM
میشه قسمت سومشو توضیح بدی که چطوری انتگرال دومی تبدیل به قسمت سوم شد؟
اون انتگرالی هم که گفتی مشکل دارم.

http://www.codecogs.com/eq.latex?%5Cint%20%5Cfrac%7B3%7D%7B%7Bx%7D%5E%7B4% 7D-1%7D=3%5Cint%20%5Cfrac%7B1%7D%7B%28%7Bx%7D%5E%7B2% 7D-1%29%28%7Bx%7D%5E%7B2%7D+1%29%7D=3%5Cint%28%5Cfrac %7B1%7D%7B2%7D%29%5Cfrac%7B%28%7Bx%7D%5E%7B2%7D+1% 29-%28%7Bx%7D%5E%7B2%7D-1%29%7D%7B%28%7Bx%7D%5E%7B2%7D-1%29%28%7Bx%7D%5E%7B2%7D+1%29%7D=
http://www.codecogs.com/eq.latex?%5Cfrac%7B3%7D%7B2%7D%5Cint%28%20%5Cfrac% 7B1%7D%7B%7Bx%7D%5E%7B2%7D-1%7D-%20%5Cfrac%7B1%7D%7B%7Bx%7D%5E%7B2%7D+1%7D%29

که از اینجا به بعدش دیگه سادس

در مورد اون یکی سوال هم داریم کافیه کسر ها رو از هم تفکیک کنیم . اینو تو پاسخ بعدی توضیح دادم

meha1368
9th November 2013, 11:54 PM
http://www.codecogs.com/eq.latex?%5Cint%20%5Cfrac%7B%7Bx%7D%5E%7B2%7D+1+3% 7D%7B%28%7Bx%7D%5E%7B2%7D-1%29%28%7Bx%7D%5E%7B2%7D+1%29%7D=%5Cint%28%20%5Cfr ac%7B%7Bx%7D%5E%7B2%7D+1%7D%7B%28%7Bx%7D%5E%7B2%7D-1%29%28%7Bx%7D%5E%7B2%7D+1%29%7D%29+%28%5Cfrac%7B3 %7D%7B%28%7Bx%7D%5E%7B2%7D-1%29%28%7Bx%7D%5E%7B2%7D+1%29%7D%29

که صورت اولی با مخرجش ساده میشه
بازم اگه نیاز به توضیح بود اطلاع بده

david77
10th November 2013, 04:43 PM
http://www.codecogs.com/eq.latex?%5Cint%20%5Cfrac%7B3%7D%7B%7Bx%7D%5E%7B4% 7D-1%7D=3%5Cint%20%5Cfrac%7B1%7D%7B%28%7Bx%7D%5E%7B2% 7D-1%29%28%7Bx%7D%5E%7B2%7D+1%29%7D=3%5Cint%28%5Cfrac %7B1%7D%7B2%7D%29%5Cfrac%7B%28%7Bx%7D%5E%7B2%7D+1% 29-%28%7Bx%7D%5E%7B2%7D-1%29%7D%7B%28%7Bx%7D%5E%7B2%7D-1%29%28%7Bx%7D%5E%7B2%7D+1%29%7D=
http://www.codecogs.com/eq.latex?%5Cfrac%7B3%7D%7B2%7D%5Cint%28%20%5Cfrac% 7B1%7D%7B%7Bx%7D%5E%7B2%7D-1%7D-%20%5Cfrac%7B1%7D%7B%7Bx%7D%5E%7B2%7D+1%7D%29



سلام خوبی؟
قسمت سومشو مشکل دارم.(صورت کسر)

meha1368
10th November 2013, 07:55 PM
سلام . تشکر
شما خوبی؟
این راه یه راه ساده س که با توجه به مخرج ، صورت رو طوری باز نویسی می کنیم که بعد از تفکیک کسر حاصل ساده تر بشه . به جای 1 صورت اون عبارت رو نوشتیم که حاصلش برابر 2 میشه و اون 1/2 رو واسه این گذاشتیم که حاصل رو یک کنه
البته راه حل اصلی این جور سوالات استفاده از روش تفکیک کسره که یه مقدار طولانی تر میشه .

david77
10th November 2013, 09:18 PM
سلام به دوستان
میخوام این معادله دیفرانسیل را از طریق جداشدنی حل کنم باید چیکار کنم؟

http://www.uc-njavan.ir/images/p1dmhspw6q8knm0bjzxn_thumb.png (http://www.uc-njavan.ir/viewer.php?file=p1dmhspw6q8knm0bjzxn.png)

meha1368
13th November 2013, 07:41 PM
سلام

http://www.codecogs.com/eq.latex?%28%7Be%7D%5E%7Bx%7D+1%29%5Cfrac%7Bdy%7D% 7Bdx%7D=y%281-%7Be%7D%5E%7Bx%7D%29%5CRightarrow%20%5Cfrac%7Bdy%7 D%7By%7D=%28%5Cfrac%7B1-%7Be%7D%5E%7Bx%7D%7D%7B%7Be%7D%5E%7Bx%7D+1%7D%29dx

که با انتگرال گیری از دو طرف جواب بدست میاد

david77
16th November 2013, 01:09 PM
سلام به دوستان

این معادله می خواستم از طریق مرتبه اول حل کنیم. باید چیکار کنم؟

http://www.uc-njavan.ir/images/6205g00sf56kmwpipt9r_thumb.png (http://www.uc-njavan.ir/viewer.php?file=6205g00sf56kmwpipt9r.png)

david77
23rd November 2013, 01:47 AM
سلام به دوستان

میخواستم برای حل این معادلات راهنمایی بفرمایید.
http://www.uc-njavan.ir/images/jvqgigyfqll739cass4_thumb.png (http://www.uc-njavan.ir/viewer.php?file=jvqgigyfqll739cass4.png)
http://www.uc-njavan.ir/images/p2bihowkm7dm1sxxlip_thumb.png (http://www.uc-njavan.ir/viewer.php?file=p2bihowkm7dm1sxxlip.png)

david77
24th February 2014, 07:15 PM
سلام به دوستان

این انتگرالو میخواستم حل کنم، میخواستم بدونم نصف اینhttp://www.uc-njavan.ir/images/e3h6j5t4697hynkg69ku_thumb.png (http://www.uc-njavan.ir/viewer.php?file=e3h6j5t4697hynkg69ku.png) چند میشه؟

http://www.uc-njavan.ir/images/p92tm71p5bmnqego7vzm_thumb.png (http://www.uc-njavan.ir/viewer.php?file=p92tm71p5bmnqego7vzm.png)

alireza2191
28th February 2014, 12:50 PM
من یه سوال دارم اگر تابع f در a مشتقپذیر باشد حاصل حد روبرو چقدر است ؟
http://www.codecogs.com/eq.latex?%5Clim%20%7B%5Cfrac%7Bx%20f%28a%29-%20a%20f%28x%29%7D%7Bx-a%7D%20%7D%20x%5Crightarrow%20a
من 2 راه حل پیدا کردم که بنظرم هر 2 صحیحه ولی 2 تا جواب مختلف داره .
راه حل اول اینه که چون تابع در a مشتقپذیره پس پیوسته است :
البته در همه این حدها x به طرف a میل میکند.


http://www.codecogs.com/eq.latex?%5Clim%20f%28x%29%20x%5Crightarrow%20a%20 =%20f%28a%29




http://www.codecogs.com/eq.latex?%5Clim%20%7B%5Cfrac%7Bx%20f%28a%29-%20a%20f%28x%29%7D%7Bx-a%7D%20%7D%20x%5Crightarrow%20a%20=%20%5Cfrac%7B%5 Clim%20x%20f%28a%29-%5Clim%20a%20f%28x%29%7D%7B%5Clim%20x-a%7D=%5Cfrac%7B%5Clim%20x%20f%28a%29-a%20f%28a%29%7D%7B%5Clim%20x-a%7D=%5Cfrac%7B%5Clim%20x%20f%28a%29-%5Clim%20a%20f%28a%29%7D%7B%5Clim%20x-a%7D=%20%5Clim%20%7B%5Cfrac%7Bx%20f%28a%29-%20a%20f%28a%29%7D%7Bx-a%7D%20%7D=%20%5Clim%20%7B%5Cfrac%7B%20f%28a%29%28 x-a%29%7D%7Bx-a%7D%20%7D=%20f%28a%29
اما راه حل دوم :



















http://www.codecogs.com/eq.latex?%5Clim%20%7B%5Cfrac%7Bx%20f%28a%29-%20a%20f%28x%29%7D%7Bx-a%7D%20%7D%20x%5Crightarrow%20a%20=%20%5Clim%20%7B %5Cfrac%7Bx%20f%28a%29-%20a%20f%28x%29-x%20f%28x%29+x%20f%28x%29%7D%7Bx-a%7D%20%7D=%5Clim%20%7B%5Cfrac%7B-x%20%28f%28x%29-%20f%28a%29%20%29%20-%20a%20f%28x%29+x%20f%28x%29%7D%7Bx-a%7D%20%7D=%5Clim%20%7B%5Cfrac%7B-x%20%28f%28x%29-%20f%28a%29%20%7D%7Bx-a%7D+%20%5Clim%20%7B%5Cfrac%7B%20x%20f%28x%29-a%20f%28x%29%20%7D%7Bx-a%7D=%20%5Clim%20-x%20%5Cacute%7Bf%20%28a%29%7D+%5Clim%20f%28x%29=%2 0f%28a%29-a%20%5Cacute%7Bf%20%28a%29%7D



راه حل اول مربوط به خودمه و راه حل دوم در یک کتاب حل المسائله - خودم فکر میکنم راه حل اول غلطه
سوالم اینه که چرا راه حل اول غلطه ؟

navid sadeghpour
2nd March 2014, 12:51 PM
من یه سوال دارم اگر تابع f در a مشتقپذیر باشد حاصل حد روبرو چقدر است ؟
http://www.codecogs.com/eq.latex?%5Clim%20%7B%5Cfrac%7Bx%20f%28a%29-%20a%20f%28x%29%7D%7Bx-a%7D%20%7D%20x%5Crightarrow%20a
من 2 راه حل پیدا کردم که بنظرم هر 2 صحیحه ولی 2 تا جواب مختلف داره .
راه حل اول اینه که چون تابع در a مشتقپذیره پس پیوسته است :
البته در همه این حدها x به طرف a میل میکند.


http://www.codecogs.com/eq.latex?%5Clim%20f%28x%29%20x%5Crightarrow%20a%20 =%20f%28a%29


http://www.codecogs.com/eq.latex?%5Clim%20%7B%5Cfrac%7Bx%20f%28a%29-%20a%20f%28x%29%7D%7Bx-a%7D%20%7D%20x%5Crightarrow%20a%20=%20%5Cfrac%7B%5 Clim%20x%20f%28a%29-%5Clim%20a%20f%28x%29%7D%7B%5Clim%20x-a%7D=%5Cfrac%7B%5Clim%20x%20f%28a%29-a%20f%28a%29%7D%7B%5Clim%20x-a%7D=%5Cfrac%7B%5Clim%20x%20f%28a%29-%5Clim%20a%20f%28a%29%7D%7B%5Clim%20x-a%7D=%20%5Clim%20%7B%5Cfrac%7Bx%20f%28a%29-%20a%20f%28a%29%7D%7Bx-a%7D%20%7D=%20%5Clim%20%7B%5Cfrac%7B%20f%28a%29%28 x-a%29%7D%7Bx-a%7D%20%7D=%20f%28a%29
اما راه حل دوم :

















http://www.codecogs.com/eq.latex?%5Clim%20%7B%5Cfrac%7Bx%20f%28a%29-%20a%20f%28x%29%7D%7Bx-a%7D%20%7D%20x%5Crightarrow%20a%20=%20%5Clim%20%7B %5Cfrac%7Bx%20f%28a%29-%20a%20f%28x%29-x%20f%28x%29+x%20f%28x%29%7D%7Bx-a%7D%20%7D=%5Clim%20%7B%5Cfrac%7B-x%20%28f%28x%29-%20f%28a%29%20%29%20-%20a%20f%28x%29+x%20f%28x%29%7D%7Bx-a%7D%20%7D=%5Clim%20%7B%5Cfrac%7B-x%20%28f%28x%29-%20f%28a%29%20%7D%7Bx-a%7D+%20%5Clim%20%7B%5Cfrac%7B%20x%20f%28x%29-a%20f%28x%29%20%7D%7Bx-a%7D=%20%5Clim%20-x%20%5Cacute%7Bf%20%28a%29%7D+%5Clim%20f%28x%29=%2 0f%28a%29-a%20%5Cacute%7Bf%20%28a%29%7D



راه حل اول مربوط به خودمه و راه حل دوم در یک کتاب حل المسائله - خودم فکر میکنم راه حل اول غلطه
سوالم اینه که چرا راه حل اول غلطه ؟
مسلمه که غلطه!شما بخشی از کسر رو به سمت a میل دادید بخشی رو ندادید!نمیشه که،مثل اینه که x تبدیل به دو متغیر جدا از هم (مستقل )شده باشه.

meha1368
31st March 2014, 01:15 PM
من یه سوال دارم اگر تابع f در a مشتقپذیر باشد حاصل حد روبرو چقدر است ؟
http://www.codecogs.com/eq.latex?%5Clim%20%7B%5Cfrac%7Bx%20f%28a%29-%20a%20f%28x%29%7D%7Bx-a%7D%20%7D%20x%5Crightarrow%20a
من 2 راه حل پیدا کردم که بنظرم هر 2 صحیحه ولی 2 تا جواب مختلف داره .
راه حل اول اینه که چون تابع در a مشتقپذیره پس پیوسته است :
البته در همه این حدها x به طرف a میل میکند.


http://www.codecogs.com/eq.latex?%5Clim%20f%28x%29%20x%5Crightarrow%20a%20 =%20f%28a%29


http://www.codecogs.com/eq.latex?%5Clim%20%7B%5Cfrac%7Bx%20f%28a%29-%20a%20f%28x%29%7D%7Bx-a%7D%20%7D%20x%5Crightarrow%20a%20=%20%5Cfrac%7B%5 Clim%20x%20f%28a%29-%5Clim%20a%20f%28x%29%7D%7B%5Clim%20x-a%7D=%5Cfrac%7B%5Clim%20x%20f%28a%29-a%20f%28a%29%7D%7B%5Clim%20x-a%7D=%5Cfrac%7B%5Clim%20x%20f%28a%29-%5Clim%20a%20f%28a%29%7D%7B%5Clim%20x-a%7D=%20%5Clim%20%7B%5Cfrac%7Bx%20f%28a%29-%20a%20f%28a%29%7D%7Bx-a%7D%20%7D=%20%5Clim%20%7B%5Cfrac%7B%20f%28a%29%28 x-a%29%7D%7Bx-a%7D%20%7D=%20f%28a%29
اما راه حل دوم :

















http://www.codecogs.com/eq.latex?%5Clim%20%7B%5Cfrac%7Bx%20f%28a%29-%20a%20f%28x%29%7D%7Bx-a%7D%20%7D%20x%5Crightarrow%20a%20=%20%5Clim%20%7B %5Cfrac%7Bx%20f%28a%29-%20a%20f%28x%29-x%20f%28x%29+x%20f%28x%29%7D%7Bx-a%7D%20%7D=%5Clim%20%7B%5Cfrac%7B-x%20%28f%28x%29-%20f%28a%29%20%29%20-%20a%20f%28x%29+x%20f%28x%29%7D%7Bx-a%7D%20%7D=%5Clim%20%7B%5Cfrac%7B-x%20%28f%28x%29-%20f%28a%29%20%7D%7Bx-a%7D+%20%5Clim%20%7B%5Cfrac%7B%20x%20f%28x%29-a%20f%28x%29%20%7D%7Bx-a%7D=%20%5Clim%20-x%20%5Cacute%7Bf%20%28a%29%7D+%5Clim%20f%28x%29=%2 0f%28a%29-a%20%5Cacute%7Bf%20%28a%29%7D



راه حل اول مربوط به خودمه و راه حل دوم در یک کتاب حل المسائله - خودم فکر میکنم راه حل اول غلطه
سوالم اینه که چرا راه حل اول غلطه ؟
حق کاملا با نویده .
به نظرم این سوال رو با هوپیتال میشه خیلی ساده تر حل کرد

david77
8th April 2014, 05:12 PM
سلام خدمت دوستان این انتگرالو تا اینجا نوشتم ولی تو بقیش مشکل دارم.

http://www.uc-njavan.ir/images/lk0pyh8ymu1mvyzl22_thumb.jpg (http://www.uc-njavan.ir/viewer.php?file=lk0pyh8ymu1mvyzl22.jpg)

s.gholizadeh
21st July 2014, 10:15 AM
با سلام
دلیل غلط بودن راه اولتون اینه که شما فقط برای f(x) جاگذاری کردید و این کاملا غلطه. همیشه حدو حساب کردنی به جای تمام x ها جاگذاری می کنیم نه یکی.

mahmoodmah
4th August 2014, 10:32 AM
سلام

راه حل رو در تصاویر ارسالی برات نوشتم
دقت کنید که احیانا خطای نداشته باشه
موفق باشید




http://www.uc-njavan.ir/images/5xqq5lulebz9nnqnhuh.jpg

http://www.uc-njavan.ir/images/n4tm2p4m7us8ujosxzl.jpg






سلام خدمت دوستان این انتگرالو تا اینجا نوشتم ولی تو بقیش مشکل دارم.

http://www.uc-njavan.ir/images/lk0pyh8ymu1mvyzl22_thumb.jpg (http://www.uc-njavan.ir/viewer.php?file=lk0pyh8ymu1mvyzl22.jpg)

sina_jackal
30th September 2014, 04:55 PM
با سلام من یه سوال خیلی ابتدایی دارم ولی الان خودم مغزم قفل شده نمیتونم پاسخ رو پیدا کنم [khejalat][negaran][negaran][gerye][gerye]
سینوس به توان 2 تتا با سینوس تتا به توان 2 چه فرقی دارن ؟

مهندس نوجوان
30th September 2014, 05:57 PM
با سلام من یه سوال خیلی ابتدایی دارم ولی الان خودم مغزم قفل شده نمیتونم پاسخ رو پیدا کنم [khejalat][negaran][negaran][gerye][gerye]
سینوس به توان 2 تتا با سینوس تتا به توان 2 چه فرقی دارن ؟
سلام فرقی ندارند....

استفاده از تمامی مطالب سایت تنها با ذکر منبع آن به نام سایت علمی نخبگان جوان و ذکر آدرس سایت مجاز است

استفاده از نام و برند نخبگان جوان به هر نحو توسط سایر سایت ها ممنوع بوده و پیگرد قانونی دارد